LSAT and Law School Admissions Forum

Get expert LSAT preparation and law school admissions advice from PowerScore Test Preparation.

User avatar
 Dave Killoran
PowerScore Staff
  • PowerScore Staff
  • Posts: 5853
  • Joined: Mar 25, 2011
|
#88233
Complete Question Explanation
(The complete setup for this game can be found here: lsat/viewtopic.php?f=174&p=88228#p88228)

The correct answer choice is (B).

The best strategy on List questions is to use the individual rules of the game to eliminate incorrect answer choices:

Answer choice (A) can be eliminated because according to the last rule when T signs with S, W must also sign with S.

Answer choice (C) can be eliminated because according to the third rule Z and Y must sign with the same agency.

Answer choice (D) can be eliminated because according to the first rule X must sign with F.

Answer choice (E) can be eliminated because according to the second rule X and Y do not sign with the same agency as each other.

Answer choice (B) is thus proven correct by process of elimination.
 ccp17
  • Posts: 6
  • Joined: Sep 14, 2016
|
#28658
Hi,

I was completing this question on of this test and while I got the Answer correct (Answer choice B) I wanted to confirm that rule 4 - "If Traugott signs with Star Agency, West also signs with Star Agency" is the one that drove this answer versus answer choice E.

I reasoned it as T&W= same agency.

However, couldn't it be interrupted as only when T=S then W=S as well and that would mean T and W can be assigned to different agencies when T is not assigned to S'?

Thanks in advance for your response.
User avatar
 Dave Killoran
PowerScore Staff
  • PowerScore Staff
  • Posts: 5853
  • Joined: Mar 25, 2011
|
#28731
Hi CCP,

Thanks for the question! I'd say that the most immediate problem with (E) is that we know from the second rule that X and Y do not sign with the same agency, yet they are both signed by Fame in this answer. So, that knocks this one out.

In reference to the fourth rule, that rule appears as:

  • TS :arrow: WS
That does not mean that T and W are a block in the game. When T is in S, they are a block, but as you noted otherwise they are free agents. This is because the rule is conditional and the sufficient condition is T in S. If, for example, T is in P, then the rule is not activated and W could be in F, P, or S.

Does that make sense? Please let me know. Thanks!
 ccp17
  • Posts: 6
  • Joined: Sep 14, 2016
|
#29803
Thank you, that did clarify my clouded mind.

Get the most out of your LSAT Prep Plus subscription.

Analyze and track your performance with our Testing and Analytics Package.